Almost uniform convergence implies convergence in measureAlmost uniform convergence of $f_n(x) = x^n$ on the interval $[0, 1]$?Uniform convergence and pointwise convergence, understandingQuantifiers in the definition of uniform convergenceDominated a.e. convergence implies almost uniform convergenceEquivalent definition of almost uniform convergenceHow to show that the given sequence of functions converges to $f$ almost everywhere, almost uniformly and in measure?Showing that the almost uniform limit of functions with bounded $L_infty$ norms is in $L_infty$Convergence in measure and almost everywhere proof

Is gzip atomic?

Why do all fields in a QFT transform like *irreducible* representations of some group?

Numbers Decrease while Letters Increase

Why would an IIS hosted site prompt for AD account credential if accessed through a hostname or IP, but not through servername?

How should I face my manager if I make a mistake because a senior coworker explained something incorrectly to me?

Heyacrazy: Careening

Why did Khan ask Admiral James T. Kirk about Project Genesis?

Architectural feasibility of a tiered circular stone keep

Why doesn't 'd /= d' throw a division by zero exception?

Are there languages that inflect adverbs for gender

What would be the challenges to taking off and landing a typical passenger jet at FL300?

I don't have the theoretical background in my PhD topic. I can't justify getting the degree

Almost uniform convergence implies convergence in measure

Examples of topos that are not ordinary spaces

Who was president of the USA?

Pythagorean triple with hypotenuse a power of 2

If all stars rotate, why was there a theory developed that requires non-rotating stars?

Why is there so little discussion / research on the philosophy of precision?

Is “I am getting married with my sister” ambiguous?

Tensorflow - logistic regrssion -oneHot Encoder - Transformed array of differt size for both train and test

Thank God it's Friday, tomorrow is THE weekend. Why the definite article?

Does norwegian.no airline overbook flights?

How do I, an introvert, communicate to my friend and only colleague, an extrovert, that I want to spend my scheduled breaks without them?

Did the British navy fail to take into account the ballistics correction due to Coriolis force during WW1 Falkland Islands battle?



Almost uniform convergence implies convergence in measure


Almost uniform convergence of $f_n(x) = x^n$ on the interval $[0, 1]$?Uniform convergence and pointwise convergence, understandingQuantifiers in the definition of uniform convergenceDominated a.e. convergence implies almost uniform convergenceEquivalent definition of almost uniform convergenceHow to show that the given sequence of functions converges to $f$ almost everywhere, almost uniformly and in measure?Showing that the almost uniform limit of functions with bounded $L_infty$ norms is in $L_infty$Convergence in measure and almost everywhere proof






.everyoneloves__top-leaderboard:empty,.everyoneloves__mid-leaderboard:empty,.everyoneloves__bot-mid-leaderboard:empty margin-bottom:0;








1












$begingroup$


Let $(A,mathcalF,mu)$ be finite measure space and $f_n$ a sequence of finite real measurable functions so that $f_nrightarrow f$ a.e. We say $f_nrightarrow f$ almost uniformly if $epsilon>0$, there is $Esubseteq A$ such that $f_n rightarrow f$ uniformly on $E^c$ and $mu(E)<epsilon$.



I want to show that $f_nrightarrow f$ almost uniformly implies convergence in $mu$. For this, suppose not. Then
$$exists eta,epsilon>0:forall Nin mathbbN:exists n>N:mu(mid f_n-fmidgeqepsilon)geq eta, $$
i.e., for infinitely many points $nin mathbbN$. From the definition of almost uniform convergence, $exists E:mu(E)<eta$ and $f_nrightarrow f$ uniformly on $E^c$. Contradiction.



Question



It seems intuitive to me. But how to deduce this contradiction precisely?
I know that if $xin E$, then it must satify the negation of uniform convergence which is
$$exists epsilon>0:forall Nin mathbbN:exists n>N:mid f_n-fmidgeqepsilon.$$
Now, $x$ may not be in $f_n text does not converge in measure to f $ if $mu(mid f_n(x)-f(x)midgeqepsilon)<eta$. So I conclude that $$f_n text does not converge in measure to f subseteq E$$ implying that $eta>mu(E)geq eta$; a contradiction.



My argument seems right but also very inefficient. How could you express this idea as clean as possible?



Thanks!










share|cite|improve this question











$endgroup$




















    1












    $begingroup$


    Let $(A,mathcalF,mu)$ be finite measure space and $f_n$ a sequence of finite real measurable functions so that $f_nrightarrow f$ a.e. We say $f_nrightarrow f$ almost uniformly if $epsilon>0$, there is $Esubseteq A$ such that $f_n rightarrow f$ uniformly on $E^c$ and $mu(E)<epsilon$.



    I want to show that $f_nrightarrow f$ almost uniformly implies convergence in $mu$. For this, suppose not. Then
    $$exists eta,epsilon>0:forall Nin mathbbN:exists n>N:mu(mid f_n-fmidgeqepsilon)geq eta, $$
    i.e., for infinitely many points $nin mathbbN$. From the definition of almost uniform convergence, $exists E:mu(E)<eta$ and $f_nrightarrow f$ uniformly on $E^c$. Contradiction.



    Question



    It seems intuitive to me. But how to deduce this contradiction precisely?
    I know that if $xin E$, then it must satify the negation of uniform convergence which is
    $$exists epsilon>0:forall Nin mathbbN:exists n>N:mid f_n-fmidgeqepsilon.$$
    Now, $x$ may not be in $f_n text does not converge in measure to f $ if $mu(mid f_n(x)-f(x)midgeqepsilon)<eta$. So I conclude that $$f_n text does not converge in measure to f subseteq E$$ implying that $eta>mu(E)geq eta$; a contradiction.



    My argument seems right but also very inefficient. How could you express this idea as clean as possible?



    Thanks!










    share|cite|improve this question











    $endgroup$
















      1












      1








      1





      $begingroup$


      Let $(A,mathcalF,mu)$ be finite measure space and $f_n$ a sequence of finite real measurable functions so that $f_nrightarrow f$ a.e. We say $f_nrightarrow f$ almost uniformly if $epsilon>0$, there is $Esubseteq A$ such that $f_n rightarrow f$ uniformly on $E^c$ and $mu(E)<epsilon$.



      I want to show that $f_nrightarrow f$ almost uniformly implies convergence in $mu$. For this, suppose not. Then
      $$exists eta,epsilon>0:forall Nin mathbbN:exists n>N:mu(mid f_n-fmidgeqepsilon)geq eta, $$
      i.e., for infinitely many points $nin mathbbN$. From the definition of almost uniform convergence, $exists E:mu(E)<eta$ and $f_nrightarrow f$ uniformly on $E^c$. Contradiction.



      Question



      It seems intuitive to me. But how to deduce this contradiction precisely?
      I know that if $xin E$, then it must satify the negation of uniform convergence which is
      $$exists epsilon>0:forall Nin mathbbN:exists n>N:mid f_n-fmidgeqepsilon.$$
      Now, $x$ may not be in $f_n text does not converge in measure to f $ if $mu(mid f_n(x)-f(x)midgeqepsilon)<eta$. So I conclude that $$f_n text does not converge in measure to f subseteq E$$ implying that $eta>mu(E)geq eta$; a contradiction.



      My argument seems right but also very inefficient. How could you express this idea as clean as possible?



      Thanks!










      share|cite|improve this question











      $endgroup$




      Let $(A,mathcalF,mu)$ be finite measure space and $f_n$ a sequence of finite real measurable functions so that $f_nrightarrow f$ a.e. We say $f_nrightarrow f$ almost uniformly if $epsilon>0$, there is $Esubseteq A$ such that $f_n rightarrow f$ uniformly on $E^c$ and $mu(E)<epsilon$.



      I want to show that $f_nrightarrow f$ almost uniformly implies convergence in $mu$. For this, suppose not. Then
      $$exists eta,epsilon>0:forall Nin mathbbN:exists n>N:mu(mid f_n-fmidgeqepsilon)geq eta, $$
      i.e., for infinitely many points $nin mathbbN$. From the definition of almost uniform convergence, $exists E:mu(E)<eta$ and $f_nrightarrow f$ uniformly on $E^c$. Contradiction.



      Question



      It seems intuitive to me. But how to deduce this contradiction precisely?
      I know that if $xin E$, then it must satify the negation of uniform convergence which is
      $$exists epsilon>0:forall Nin mathbbN:exists n>N:mid f_n-fmidgeqepsilon.$$
      Now, $x$ may not be in $f_n text does not converge in measure to f $ if $mu(mid f_n(x)-f(x)midgeqepsilon)<eta$. So I conclude that $$f_n text does not converge in measure to f subseteq E$$ implying that $eta>mu(E)geq eta$; a contradiction.



      My argument seems right but also very inefficient. How could you express this idea as clean as possible?



      Thanks!







      real-analysis measure-theory proof-writing






      share|cite|improve this question















      share|cite|improve this question













      share|cite|improve this question




      share|cite|improve this question








      edited 7 hours ago









      Gabriel Romon

      19.2k5 gold badges37 silver badges88 bronze badges




      19.2k5 gold badges37 silver badges88 bronze badges










      asked 8 hours ago









      DanmatDanmat

      41610 bronze badges




      41610 bronze badges























          2 Answers
          2






          active

          oldest

          votes


















          4













          $begingroup$

          Here's a way without going for contradiction: Let $epsilon >0$ be fixed and consider some $delta >0$.



          By almost uniform convergence, there exists some $E$ with $mu(E)leq delta$ and some $N$ such that $ngeq Nimplies forall xin E^c, |f_n(x)-f(x)|< epsilon$.



          For $ngeq N$, note the inclusion $(|f_n-f|geq epsilon) subset E$, hence $mu((|f_n-f|geq epsilon))leq mu(E)leq delta$.



          Hence $forall delta>0, exists N, ngeq N implies mu((|f_n-f|geq epsilon))leq delta$. Thus $ mu((|f_n-f|geq epsilon)) to 0$.






          share|cite|improve this answer











          $endgroup$














          • $begingroup$
            Thanks! The inclusion $mid f_n-fmidgeq epsilonsubset E$ that I don't understand. What is this infinity-norm that you wrote?
            $endgroup$
            – Danmat
            8 hours ago










          • $begingroup$
            @Danmat I removed the infinity norm (which denotes the sup-norm), do you understand now ?
            $endgroup$
            – Gabriel Romon
            8 hours ago










          • $begingroup$
            It has something to do with the fact that uniform convergence implies pointwise convergence? And then the complement of the latter is contained in the complement of the former.
            $endgroup$
            – Danmat
            7 hours ago











          • $begingroup$
            @Danmat I just wrote the definition of uniform convergence on $E^c$. Pointwise convergence is not used in the proof.
            $endgroup$
            – Gabriel Romon
            7 hours ago










          • $begingroup$
            But, $E$ is constituted of elements in $A$ such that $f_n rightarrow f$ does not converge uniformly, right?
            $endgroup$
            – Danmat
            7 hours ago



















          2













          $begingroup$

          This is proved more succinctly by a direct proof.



          Given $varepsilon > 0,$ let $E$ be as in the definition of almost-uniform convergence. Then there is some $N$ such that $n geq N$ implies
          $$mu(E^c cap ) = 0$$
          What does this imply for $mu(|f_n-f|geq varepsilon)?$






          share|cite|improve this answer









          $endgroup$














          • $begingroup$
            How to conclude that $mid f_n-fmid subseteq E$?
            $endgroup$
            – Danmat
            7 hours ago










          • $begingroup$
            @Danmat Forget about $E$ for a second. On $E^c,$ the functions converge uniformly. This means that there will be some $N$ such that $n geq N$ implies $|f_n - f| < varepsilon.$ In particular, on $E^c,$ $$n geq N implies geq varepsilon = emptyset.$$ This directly means $$E^c cap geq varepsilon = emptyset.$$ If you want, then you can use the property of sets that $$A^c cap B = emptyset iff B subseteq A$$
            $endgroup$
            – Brian Moehring
            7 hours ago











          • $begingroup$
            Very nice! Now, it's all clear to me!
            $endgroup$
            – Danmat
            7 hours ago













          Your Answer








          StackExchange.ready(function()
          var channelOptions =
          tags: "".split(" "),
          id: "69"
          ;
          initTagRenderer("".split(" "), "".split(" "), channelOptions);

          StackExchange.using("externalEditor", function()
          // Have to fire editor after snippets, if snippets enabled
          if (StackExchange.settings.snippets.snippetsEnabled)
          StackExchange.using("snippets", function()
          createEditor();
          );

          else
          createEditor();

          );

          function createEditor()
          StackExchange.prepareEditor(
          heartbeatType: 'answer',
          autoActivateHeartbeat: false,
          convertImagesToLinks: true,
          noModals: true,
          showLowRepImageUploadWarning: true,
          reputationToPostImages: 10,
          bindNavPrevention: true,
          postfix: "",
          imageUploader:
          brandingHtml: "Powered by u003ca class="icon-imgur-white" href="https://imgur.com/"u003eu003c/au003e",
          contentPolicyHtml: "User contributions licensed under u003ca href="https://creativecommons.org/licenses/by-sa/3.0/"u003ecc by-sa 3.0 with attribution requiredu003c/au003e u003ca href="https://stackoverflow.com/legal/content-policy"u003e(content policy)u003c/au003e",
          allowUrls: true
          ,
          noCode: true, onDemand: true,
          discardSelector: ".discard-answer"
          ,immediatelyShowMarkdownHelp:true
          );



          );













          draft saved

          draft discarded


















          StackExchange.ready(
          function ()
          StackExchange.openid.initPostLogin('.new-post-login', 'https%3a%2f%2fmath.stackexchange.com%2fquestions%2f3332265%2falmost-uniform-convergence-implies-convergence-in-measure%23new-answer', 'question_page');

          );

          Post as a guest















          Required, but never shown

























          2 Answers
          2






          active

          oldest

          votes








          2 Answers
          2






          active

          oldest

          votes









          active

          oldest

          votes






          active

          oldest

          votes









          4













          $begingroup$

          Here's a way without going for contradiction: Let $epsilon >0$ be fixed and consider some $delta >0$.



          By almost uniform convergence, there exists some $E$ with $mu(E)leq delta$ and some $N$ such that $ngeq Nimplies forall xin E^c, |f_n(x)-f(x)|< epsilon$.



          For $ngeq N$, note the inclusion $(|f_n-f|geq epsilon) subset E$, hence $mu((|f_n-f|geq epsilon))leq mu(E)leq delta$.



          Hence $forall delta>0, exists N, ngeq N implies mu((|f_n-f|geq epsilon))leq delta$. Thus $ mu((|f_n-f|geq epsilon)) to 0$.






          share|cite|improve this answer











          $endgroup$














          • $begingroup$
            Thanks! The inclusion $mid f_n-fmidgeq epsilonsubset E$ that I don't understand. What is this infinity-norm that you wrote?
            $endgroup$
            – Danmat
            8 hours ago










          • $begingroup$
            @Danmat I removed the infinity norm (which denotes the sup-norm), do you understand now ?
            $endgroup$
            – Gabriel Romon
            8 hours ago










          • $begingroup$
            It has something to do with the fact that uniform convergence implies pointwise convergence? And then the complement of the latter is contained in the complement of the former.
            $endgroup$
            – Danmat
            7 hours ago











          • $begingroup$
            @Danmat I just wrote the definition of uniform convergence on $E^c$. Pointwise convergence is not used in the proof.
            $endgroup$
            – Gabriel Romon
            7 hours ago










          • $begingroup$
            But, $E$ is constituted of elements in $A$ such that $f_n rightarrow f$ does not converge uniformly, right?
            $endgroup$
            – Danmat
            7 hours ago
















          4













          $begingroup$

          Here's a way without going for contradiction: Let $epsilon >0$ be fixed and consider some $delta >0$.



          By almost uniform convergence, there exists some $E$ with $mu(E)leq delta$ and some $N$ such that $ngeq Nimplies forall xin E^c, |f_n(x)-f(x)|< epsilon$.



          For $ngeq N$, note the inclusion $(|f_n-f|geq epsilon) subset E$, hence $mu((|f_n-f|geq epsilon))leq mu(E)leq delta$.



          Hence $forall delta>0, exists N, ngeq N implies mu((|f_n-f|geq epsilon))leq delta$. Thus $ mu((|f_n-f|geq epsilon)) to 0$.






          share|cite|improve this answer











          $endgroup$














          • $begingroup$
            Thanks! The inclusion $mid f_n-fmidgeq epsilonsubset E$ that I don't understand. What is this infinity-norm that you wrote?
            $endgroup$
            – Danmat
            8 hours ago










          • $begingroup$
            @Danmat I removed the infinity norm (which denotes the sup-norm), do you understand now ?
            $endgroup$
            – Gabriel Romon
            8 hours ago










          • $begingroup$
            It has something to do with the fact that uniform convergence implies pointwise convergence? And then the complement of the latter is contained in the complement of the former.
            $endgroup$
            – Danmat
            7 hours ago











          • $begingroup$
            @Danmat I just wrote the definition of uniform convergence on $E^c$. Pointwise convergence is not used in the proof.
            $endgroup$
            – Gabriel Romon
            7 hours ago










          • $begingroup$
            But, $E$ is constituted of elements in $A$ such that $f_n rightarrow f$ does not converge uniformly, right?
            $endgroup$
            – Danmat
            7 hours ago














          4














          4










          4







          $begingroup$

          Here's a way without going for contradiction: Let $epsilon >0$ be fixed and consider some $delta >0$.



          By almost uniform convergence, there exists some $E$ with $mu(E)leq delta$ and some $N$ such that $ngeq Nimplies forall xin E^c, |f_n(x)-f(x)|< epsilon$.



          For $ngeq N$, note the inclusion $(|f_n-f|geq epsilon) subset E$, hence $mu((|f_n-f|geq epsilon))leq mu(E)leq delta$.



          Hence $forall delta>0, exists N, ngeq N implies mu((|f_n-f|geq epsilon))leq delta$. Thus $ mu((|f_n-f|geq epsilon)) to 0$.






          share|cite|improve this answer











          $endgroup$



          Here's a way without going for contradiction: Let $epsilon >0$ be fixed and consider some $delta >0$.



          By almost uniform convergence, there exists some $E$ with $mu(E)leq delta$ and some $N$ such that $ngeq Nimplies forall xin E^c, |f_n(x)-f(x)|< epsilon$.



          For $ngeq N$, note the inclusion $(|f_n-f|geq epsilon) subset E$, hence $mu((|f_n-f|geq epsilon))leq mu(E)leq delta$.



          Hence $forall delta>0, exists N, ngeq N implies mu((|f_n-f|geq epsilon))leq delta$. Thus $ mu((|f_n-f|geq epsilon)) to 0$.







          share|cite|improve this answer














          share|cite|improve this answer



          share|cite|improve this answer








          edited 8 hours ago

























          answered 8 hours ago









          Gabriel RomonGabriel Romon

          19.2k5 gold badges37 silver badges88 bronze badges




          19.2k5 gold badges37 silver badges88 bronze badges














          • $begingroup$
            Thanks! The inclusion $mid f_n-fmidgeq epsilonsubset E$ that I don't understand. What is this infinity-norm that you wrote?
            $endgroup$
            – Danmat
            8 hours ago










          • $begingroup$
            @Danmat I removed the infinity norm (which denotes the sup-norm), do you understand now ?
            $endgroup$
            – Gabriel Romon
            8 hours ago










          • $begingroup$
            It has something to do with the fact that uniform convergence implies pointwise convergence? And then the complement of the latter is contained in the complement of the former.
            $endgroup$
            – Danmat
            7 hours ago











          • $begingroup$
            @Danmat I just wrote the definition of uniform convergence on $E^c$. Pointwise convergence is not used in the proof.
            $endgroup$
            – Gabriel Romon
            7 hours ago










          • $begingroup$
            But, $E$ is constituted of elements in $A$ such that $f_n rightarrow f$ does not converge uniformly, right?
            $endgroup$
            – Danmat
            7 hours ago

















          • $begingroup$
            Thanks! The inclusion $mid f_n-fmidgeq epsilonsubset E$ that I don't understand. What is this infinity-norm that you wrote?
            $endgroup$
            – Danmat
            8 hours ago










          • $begingroup$
            @Danmat I removed the infinity norm (which denotes the sup-norm), do you understand now ?
            $endgroup$
            – Gabriel Romon
            8 hours ago










          • $begingroup$
            It has something to do with the fact that uniform convergence implies pointwise convergence? And then the complement of the latter is contained in the complement of the former.
            $endgroup$
            – Danmat
            7 hours ago











          • $begingroup$
            @Danmat I just wrote the definition of uniform convergence on $E^c$. Pointwise convergence is not used in the proof.
            $endgroup$
            – Gabriel Romon
            7 hours ago










          • $begingroup$
            But, $E$ is constituted of elements in $A$ such that $f_n rightarrow f$ does not converge uniformly, right?
            $endgroup$
            – Danmat
            7 hours ago
















          $begingroup$
          Thanks! The inclusion $mid f_n-fmidgeq epsilonsubset E$ that I don't understand. What is this infinity-norm that you wrote?
          $endgroup$
          – Danmat
          8 hours ago




          $begingroup$
          Thanks! The inclusion $mid f_n-fmidgeq epsilonsubset E$ that I don't understand. What is this infinity-norm that you wrote?
          $endgroup$
          – Danmat
          8 hours ago












          $begingroup$
          @Danmat I removed the infinity norm (which denotes the sup-norm), do you understand now ?
          $endgroup$
          – Gabriel Romon
          8 hours ago




          $begingroup$
          @Danmat I removed the infinity norm (which denotes the sup-norm), do you understand now ?
          $endgroup$
          – Gabriel Romon
          8 hours ago












          $begingroup$
          It has something to do with the fact that uniform convergence implies pointwise convergence? And then the complement of the latter is contained in the complement of the former.
          $endgroup$
          – Danmat
          7 hours ago





          $begingroup$
          It has something to do with the fact that uniform convergence implies pointwise convergence? And then the complement of the latter is contained in the complement of the former.
          $endgroup$
          – Danmat
          7 hours ago













          $begingroup$
          @Danmat I just wrote the definition of uniform convergence on $E^c$. Pointwise convergence is not used in the proof.
          $endgroup$
          – Gabriel Romon
          7 hours ago




          $begingroup$
          @Danmat I just wrote the definition of uniform convergence on $E^c$. Pointwise convergence is not used in the proof.
          $endgroup$
          – Gabriel Romon
          7 hours ago












          $begingroup$
          But, $E$ is constituted of elements in $A$ such that $f_n rightarrow f$ does not converge uniformly, right?
          $endgroup$
          – Danmat
          7 hours ago





          $begingroup$
          But, $E$ is constituted of elements in $A$ such that $f_n rightarrow f$ does not converge uniformly, right?
          $endgroup$
          – Danmat
          7 hours ago














          2













          $begingroup$

          This is proved more succinctly by a direct proof.



          Given $varepsilon > 0,$ let $E$ be as in the definition of almost-uniform convergence. Then there is some $N$ such that $n geq N$ implies
          $$mu(E^c cap ) = 0$$
          What does this imply for $mu(|f_n-f|geq varepsilon)?$






          share|cite|improve this answer









          $endgroup$














          • $begingroup$
            How to conclude that $mid f_n-fmid subseteq E$?
            $endgroup$
            – Danmat
            7 hours ago










          • $begingroup$
            @Danmat Forget about $E$ for a second. On $E^c,$ the functions converge uniformly. This means that there will be some $N$ such that $n geq N$ implies $|f_n - f| < varepsilon.$ In particular, on $E^c,$ $$n geq N implies geq varepsilon = emptyset.$$ This directly means $$E^c cap geq varepsilon = emptyset.$$ If you want, then you can use the property of sets that $$A^c cap B = emptyset iff B subseteq A$$
            $endgroup$
            – Brian Moehring
            7 hours ago











          • $begingroup$
            Very nice! Now, it's all clear to me!
            $endgroup$
            – Danmat
            7 hours ago















          2













          $begingroup$

          This is proved more succinctly by a direct proof.



          Given $varepsilon > 0,$ let $E$ be as in the definition of almost-uniform convergence. Then there is some $N$ such that $n geq N$ implies
          $$mu(E^c cap ) = 0$$
          What does this imply for $mu(|f_n-f|geq varepsilon)?$






          share|cite|improve this answer









          $endgroup$














          • $begingroup$
            How to conclude that $mid f_n-fmid subseteq E$?
            $endgroup$
            – Danmat
            7 hours ago










          • $begingroup$
            @Danmat Forget about $E$ for a second. On $E^c,$ the functions converge uniformly. This means that there will be some $N$ such that $n geq N$ implies $|f_n - f| < varepsilon.$ In particular, on $E^c,$ $$n geq N implies geq varepsilon = emptyset.$$ This directly means $$E^c cap geq varepsilon = emptyset.$$ If you want, then you can use the property of sets that $$A^c cap B = emptyset iff B subseteq A$$
            $endgroup$
            – Brian Moehring
            7 hours ago











          • $begingroup$
            Very nice! Now, it's all clear to me!
            $endgroup$
            – Danmat
            7 hours ago













          2














          2










          2







          $begingroup$

          This is proved more succinctly by a direct proof.



          Given $varepsilon > 0,$ let $E$ be as in the definition of almost-uniform convergence. Then there is some $N$ such that $n geq N$ implies
          $$mu(E^c cap ) = 0$$
          What does this imply for $mu(|f_n-f|geq varepsilon)?$






          share|cite|improve this answer









          $endgroup$



          This is proved more succinctly by a direct proof.



          Given $varepsilon > 0,$ let $E$ be as in the definition of almost-uniform convergence. Then there is some $N$ such that $n geq N$ implies
          $$mu(E^c cap ) = 0$$
          What does this imply for $mu(|f_n-f|geq varepsilon)?$







          share|cite|improve this answer












          share|cite|improve this answer



          share|cite|improve this answer










          answered 8 hours ago









          Brian MoehringBrian Moehring

          1,6192 silver badges9 bronze badges




          1,6192 silver badges9 bronze badges














          • $begingroup$
            How to conclude that $mid f_n-fmid subseteq E$?
            $endgroup$
            – Danmat
            7 hours ago










          • $begingroup$
            @Danmat Forget about $E$ for a second. On $E^c,$ the functions converge uniformly. This means that there will be some $N$ such that $n geq N$ implies $|f_n - f| < varepsilon.$ In particular, on $E^c,$ $$n geq N implies geq varepsilon = emptyset.$$ This directly means $$E^c cap geq varepsilon = emptyset.$$ If you want, then you can use the property of sets that $$A^c cap B = emptyset iff B subseteq A$$
            $endgroup$
            – Brian Moehring
            7 hours ago











          • $begingroup$
            Very nice! Now, it's all clear to me!
            $endgroup$
            – Danmat
            7 hours ago
















          • $begingroup$
            How to conclude that $mid f_n-fmid subseteq E$?
            $endgroup$
            – Danmat
            7 hours ago










          • $begingroup$
            @Danmat Forget about $E$ for a second. On $E^c,$ the functions converge uniformly. This means that there will be some $N$ such that $n geq N$ implies $|f_n - f| < varepsilon.$ In particular, on $E^c,$ $$n geq N implies geq varepsilon = emptyset.$$ This directly means $$E^c cap geq varepsilon = emptyset.$$ If you want, then you can use the property of sets that $$A^c cap B = emptyset iff B subseteq A$$
            $endgroup$
            – Brian Moehring
            7 hours ago











          • $begingroup$
            Very nice! Now, it's all clear to me!
            $endgroup$
            – Danmat
            7 hours ago















          $begingroup$
          How to conclude that $mid f_n-fmid subseteq E$?
          $endgroup$
          – Danmat
          7 hours ago




          $begingroup$
          How to conclude that $mid f_n-fmid subseteq E$?
          $endgroup$
          – Danmat
          7 hours ago












          $begingroup$
          @Danmat Forget about $E$ for a second. On $E^c,$ the functions converge uniformly. This means that there will be some $N$ such that $n geq N$ implies $|f_n - f| < varepsilon.$ In particular, on $E^c,$ $$n geq N implies geq varepsilon = emptyset.$$ This directly means $$E^c cap geq varepsilon = emptyset.$$ If you want, then you can use the property of sets that $$A^c cap B = emptyset iff B subseteq A$$
          $endgroup$
          – Brian Moehring
          7 hours ago





          $begingroup$
          @Danmat Forget about $E$ for a second. On $E^c,$ the functions converge uniformly. This means that there will be some $N$ such that $n geq N$ implies $|f_n - f| < varepsilon.$ In particular, on $E^c,$ $$n geq N implies geq varepsilon = emptyset.$$ This directly means $$E^c cap geq varepsilon = emptyset.$$ If you want, then you can use the property of sets that $$A^c cap B = emptyset iff B subseteq A$$
          $endgroup$
          – Brian Moehring
          7 hours ago













          $begingroup$
          Very nice! Now, it's all clear to me!
          $endgroup$
          – Danmat
          7 hours ago




          $begingroup$
          Very nice! Now, it's all clear to me!
          $endgroup$
          – Danmat
          7 hours ago

















          draft saved

          draft discarded
















































          Thanks for contributing an answer to Mathematics Stack Exchange!


          • Please be sure to answer the question. Provide details and share your research!

          But avoid


          • Asking for help, clarification, or responding to other answers.

          • Making statements based on opinion; back them up with references or personal experience.

          Use MathJax to format equations. MathJax reference.


          To learn more, see our tips on writing great answers.




          draft saved


          draft discarded














          StackExchange.ready(
          function ()
          StackExchange.openid.initPostLogin('.new-post-login', 'https%3a%2f%2fmath.stackexchange.com%2fquestions%2f3332265%2falmost-uniform-convergence-implies-convergence-in-measure%23new-answer', 'question_page');

          );

          Post as a guest















          Required, but never shown





















































          Required, but never shown














          Required, but never shown












          Required, but never shown







          Required, but never shown

































          Required, but never shown














          Required, but never shown












          Required, but never shown







          Required, but never shown







          Popular posts from this blog

          Invision Community Contents History See also References External links Navigation menuProprietaryinvisioncommunity.comIPS Community ForumsIPS Community Forumsthis blog entry"License Changes, IP.Board 3.4, and the Future""Interview -- Matt Mecham of Ibforums""CEO Invision Power Board, Matt Mecham Is a Liar, Thief!"IPB License Explanation 1.3, 1.3.1, 2.0, and 2.1ArchivedSecurity Fixes, Updates And Enhancements For IPB 1.3.1Archived"New Demo Accounts - Invision Power Services"the original"New Default Skin"the original"Invision Power Board 3.0.0 and Applications Released"the original"Archived copy"the original"Perpetual licenses being done away with""Release Notes - Invision Power Services""Introducing: IPS Community Suite 4!"Invision Community Release Notes

          Canceling a color specificationRandomly assigning color to Graphics3D objects?Default color for Filling in Mathematica 9Coloring specific elements of sets with a prime modified order in an array plotHow to pick a color differing significantly from the colors already in a given color list?Detection of the text colorColor numbers based on their valueCan color schemes for use with ColorData include opacity specification?My dynamic color schemes

          Tom Holland Mục lục Đầu đời và giáo dục | Sự nghiệp | Cuộc sống cá nhân | Phim tham gia | Giải thưởng và đề cử | Chú thích | Liên kết ngoài | Trình đơn chuyển hướngProfile“Person Details for Thomas Stanley Holland, "England and Wales Birth Registration Index, 1837-2008" — FamilySearch.org”"Meet Tom Holland... the 16-year-old star of The Impossible""Schoolboy actor Tom Holland finds himself in Oscar contention for role in tsunami drama"“Naomi Watts on the Prince William and Harry's reaction to her film about the late Princess Diana”lưu trữ"Holland and Pflueger Are West End's Two New 'Billy Elliots'""I'm so envious of my son, the movie star! British writer Dominic Holland's spent 20 years trying to crack Hollywood - but he's been beaten to it by a very unlikely rival"“Richard and Margaret Povey of Jersey, Channel Islands, UK: Information about Thomas Stanley Holland”"Tom Holland to play Billy Elliot""New Billy Elliot leaving the garage"Billy Elliot the Musical - Tom Holland - Billy"A Tale of four Billys: Tom Holland""The Feel Good Factor""Thames Christian College schoolboys join Myleene Klass for The Feelgood Factor""Government launches £600,000 arts bursaries pilot""BILLY's Chapman, Holland, Gardner & Jackson-Keen Visit Prime Minister""Elton John 'blown away' by Billy Elliot fifth birthday" (video with John's interview and fragments of Holland's performance)"First News interviews Arrietty's Tom Holland"“33rd Critics' Circle Film Awards winners”“National Board of Review Current Awards”Bản gốc"Ron Howard Whaling Tale 'In The Heart Of The Sea' Casts Tom Holland"“'Spider-Man' Finds Tom Holland to Star as New Web-Slinger”lưu trữ“Captain America: Civil War (2016)”“Film Review: ‘Captain America: Civil War’”lưu trữ“‘Captain America: Civil War’ review: Choose your own avenger”lưu trữ“The Lost City of Z reviews”“Sony Pictures and Marvel Studios Find Their 'Spider-Man' Star and Director”“‘Mary Magdalene’, ‘Current War’ & ‘Wind River’ Get 2017 Release Dates From Weinstein”“Lionsgate Unleashing Daisy Ridley & Tom Holland Starrer ‘Chaos Walking’ In Cannes”“PTA's 'Master' Leads Chicago Film Critics Nominations, UPDATED: Houston and Indiana Critics Nominations”“Nominaciones Goya 2013 Telecinco Cinema – ENG”“Jameson Empire Film Awards: Martin Freeman wins best actor for performance in The Hobbit”“34th Annual Young Artist Awards”Bản gốc“Teen Choice Awards 2016—Captain America: Civil War Leads Second Wave of Nominations”“BAFTA Film Award Nominations: ‘La La Land’ Leads Race”“Saturn Awards Nominations 2017: 'Rogue One,' 'Walking Dead' Lead”Tom HollandTom HollandTom HollandTom Hollandmedia.gettyimages.comWorldCat Identities300279794no20130442900000 0004 0355 42791085670554170004732cb16706349t(data)XX5557367